You are on page 1of 5

Math 114 Problem Set 2

Roman Berens
with collaboration from George Torres

September 16, 2014

1. Prove that every closed set F R is a G set. (That is, F is a countable intersection of
open sets.)
For each a F , let

[
1
1
Un =
a ,a +
.
n
n
aF

We see that each Un is open, as it is the union of open intervals. We see that F Un

T
T
for each n N, so we see F
Un . To show that F
Un , we shall show that
n=1

n=1

FC

if x
/ F, x
/
Un . We note that
is open, so there exists some n N such that

n=1


1
1
1
1
x ,x +
F = . This means that for all a F , a
/ x ,x +
, which implies
n
n
n
n



T
T
1
1
that for all a F , x
/ a ,a +
. Thus x
/ Un , so x
/
Un . Thus F =
Un , so
n
n
n=1
n=1
any closed set is a countable intersection of open sets.
2. Let E P
= Q [0, 1] and let (In ) be a finite collection of open intervals covering E.
Show that
`(In ) 1
Let {In }N
n=1 be our finite collection that covers E. We shall show that
!
N
[
[0, 1] A
In ,
n=1
N
S

where A is a set of measure zero. If [0, 1]

In , then the above is true, as A = is a set of

n=1
N
S

measure zero. We thus assume that [0, 1] 6

In . This means that there is some irrational

n=1

number x A, x

N
S

In . Since Q is dense in R, x must be isolated in A. This can only

n=1

occur if the intervals (a, x) and (x, b) are in the collection of intervals. Since we can associate
every element of A to two distinct intervals in the finite collection, A must also be finite.
Thus A has measure zero. It then follows that
!
N
[
m([0, 1]) = m(A) + m
In
(monotonicty)
n=1

=m

N
[

!
In

(definition of A)

` (In ) .

(finite subadditivity)

n=1

N
X
n=1

Thus

N
P

` (In ) 1.

n=1

3. Prove that Q is not a G set. (Hint: Let Q = {p1 , p2 , . . .}) and construct a nested
sequence of compact intervals K1 K2 such that Ki Ui {pi }.)
Assume for the sake of contradiction that Q is a G set. Then there is a collection of

T
open sets {Ui } such that Q =
Ui . We then define Ui0 = Ui = {pi }. We see that
i=1

\
i=1

Ui0 =

(Ui {pi }) =

i=1

Ui

i=1

{pi } = Q Q = .

i=1

We now construct the compact nested intervals Ki in the following way. We choose some
open interval (, ) U1 such that p1
/ (, ) and p2 (, ). We see that this interval
is a subset of U10 . We then define a nonempty compact interval K1 = [0 , 0 ] (, ). We
define K1 as this compact interval. Now, given Ki = [a, b], to construct the interval Ki+1 , we
pick an ` Ki Ui+1 , ` 6= pi+1 . We then choose an interval (, ) Ki Ui+1 such that
` (, ) and pi+1
/ (, ). We then define Ki+1 = [0 , 0 ] (, ).
We have thus constructed a nested sequence of compact intervals. We see that Ki Ui0 . It
then follows that
!
!

\
\
0
Ki
Ui = .
i=1

i=1

Thus we have a descended countable collection of nonempty compact sets of real numbers
whose intersection is empty. This contradicts the Nested Set Theorem. Thus Q is not a G set.
4. Let f : R R be an arbitrary function. Show that the set of points C R where f is
continuous is a G set. Is there a function f (x) which is continuous at the rational numbers
and discontinuous at the irrational numbers?
We define the set
Ue = {x R |  > e, > 0 such that |x y| < = |f (x) f (y)| < }.
These sets define the points at which f is quasi-continuous, i.e., they satisfy the definition
of continuity for all  > e. Letting ei = 1/i for i N, we see that
\
Uei = U0 ,
ei

which is C, the set of points on which f is continuous. We now must show that each Uei is
open, for then we have that C is a countable union of open sets (a G set).
We consider a particular Uei . For each  > ei , we let  > 0 be the largest that satisfies the above definition. We then let 0 be the infimum of the set of such  . We then
consider the interval (x  , x +  ) . By the above definition, f is continuous at every point
in this interval. Thus we have found a neighborhood around x in Uei for all x Uei . Thus
Ue is open, and thus C is a G set.
We see that there can be no function that is continuous on the rational numbers but
not the irrational numbers because Q is not a G set, as shown in part (3).

5. Let hfn i be a sequence of continuous functions defined on R. Show that the set E
of points where fn (x) converges is a F . (That is, E is a countable intersection of F sets.
An F is a countable union of closed sets.)
Let U = {x R | N N such that m, n > N, |fm (x) fn (x)| }. Letting

T
k = 1/k for k N, we see that the points where the sequence converges is Uk . We can
k

see that for each k , Uk can be written as a union of UNk over the values of N N, where
the UNk are defined as
UNk = {x R | m, n > N, |fn (x) fm (x)| k }
We shall show that these sets are closed. Let x be an element of UNk
such that
|fn (x) fm (x)| k > 2 > 0.

C

We choose > 0

We then find n such that for all a (x n , x + n ), |fn (a) fn (x)| < . We define m
similarly and let = min(n , m ). We then see
|fn (a) fn (x)| + |fm (a) fm (x)| < 2.
It then follows that
|fn (a) fn (x)| + |fm (a) fm (x)| < |fn (x) fm (x)| k .
By the triangle inequality,
k < |fn (x) fm (x)| |fn (a) fn (x)| |fm (a) fm (x)| |fn (a) fm (a)|.
Thus the sequence of functions converges for every point a (x n , x + n ). Since this holds
C
for all x UNk , the set is open. Thus (UNk is closed and Uk is the countable union of
closed sets (an F set)for each k . Then the points where the sequence of functions converges
is a countable intersection of F sets (an F set).
6. Give an example of a point x in the Cantor set K which does not lie on the boundary of one of the complementary intervals (a, b) R K.
Any point in the Cantor set has a ternary expansion that consists of zeroes and twos.
The points that lie on the boundaries of one of the complementary intervals are those that
also have another ternary representation that has a one. For example, 1/3 can be written as
.13 or .0222 . . .3 . Thus a point in the Cantor set that does not lie on the boundary of one of
the complementary intervals will only have a ternary representation with zeroes or twos. For
example, 1/4 = .020202 . . .3 is one such number.
7. Give an example of a closed set F [0, 1] with positive measure but empty interior. (Hint:
mimic the construction of the Cantor set, removing intervals of length (1/2)3n at stage n.)
1
At stage 1, we remove an interval of length . At stage 2, we remove 2 intervals of length
6
1
1
. At stage n, we remove n intervals of length
. Thus the total length of the removed
18
2 3n
intervals is

 
X
2n2
1X 2 n 1
1
=
= 2= .
n
3
4
3
4
2
n=1

n=1

Since the complement of F in [0, 1] is the union of disjoint open intervals, its measure is
1
simply the sum of the lengths of the intervals. Thus m ([0, 1] F ) = . By the excision
2
property,
1
1
m(F ) = m ([0, 1]) m ([0, 1] F ) = 1 = .
2
2
We see that F is closed since it is the complement of a countable union of open sets, which is
necessarily open. F has an empty interior because it contains no intervals.
8. Let A [0, 1] be a set of positive measure. Show that the set
A A = {x y | x, y A}
contains a nonempty open interval.
Since A has positive finite measure, there is a closed set F and and open set U such
that
F A U,
m(F ) +  > A > m(U ) 
for all . We choose F and U such that 2m(F ) > m(U ). We then choose a open set V around
0 such that F + V U . We let v V and suppose that (F + v) F = . We then see that
m(F + v) + m(F ) = 2m(F ) < m(U ),
which contradicts out choice of F and U . Thus we see that for all v V , (F + v) F =
6 ,
which means that there are two points a1 , a2 F that map to each other under the translate
of F by v, i.e., a1 + v = a2 . This implies that V A A, so A A contains a nonempty
open interval.

You might also like